LSAT and Law School Admissions Forum

Get expert LSAT preparation and law school admissions advice from PowerScore Test Preparation.

User avatar
 Dave Killoran
PowerScore Staff
  • PowerScore Staff
  • Posts: 5853
  • Joined: Mar 25, 2011
|
#88995
Complete Question Explanation
(The complete setup for this game can be found here: lsat/viewtopic.php?f=237&p=88987)

The correct answer choice is (C)

In this List question, you can still apply each rule to the answers, but you must be careful with the first rule. Because each answer includes just four consecutive people, exactly one of F or G must appear in the correct choice, a fact the test makers prey upon in the first two answers.

Answer choice (A): This answer choice violates the first rule. Because both F and G are included, they are not sitting opposite of each other.

Answer choice (B): This answer choice violates the first rule. Because neither F nor G are included, they are again not sitting opposite of each other.

Answer choice (C): This is the correct answer choice.

Answer choice (D): This answer violates the third rule.

Answer choice (E): This answer violates the second rule.

Get the most out of your LSAT Prep Plus subscription.

Analyze and track your performance with our Testing and Analytics Package.